Question

Create an equally-weighted index for period 1, w/ initial value of 3000. How many shares of...

Create an equally-weighted index for period 1, w/ initial value of 3000.

  1. How many shares of A, B and C you need to purchase in order to create this index at time 1?

  2. What is the value of the equally-weighted index at time 2?

  3. Rebalance the index at time 2; what will be the number of shares of each A, B and C that you need to have after rebalancing and how you should trade each one of them?

Stock

P1

Q1

P2

Q2

P3

Q3

A

$70

200

$72

200

$37.5

400

B

$85

500

$81

500

$83

500

C

$105

300

$98

300

$105.5

300

0 0
Add a comment Improve this question Transcribed image text
Answer #1

For an equal weighted index we need to invest same amount in each of the three stocks.

Since initial value = 3000 we will have to start initially by investing 1000 in each of the stocks A, B and C.

Now the prices are given in the table and as per that we will buy

1000/70 = 14.29 shares of A

1000/85 = 11.76 shares of B

1000/705 = 9.52 shares of C

At time 2, the prices are given in the table as per that the values of each stocks are

A = $72 * 14.29 = 1028.88

B = $81 * 11.76 = 952.56

C = $98 * 9.52 = 932.96

Value of equal weighted index = 1028.88 + 952.56 + 932.96 = 2914.4

Rebalancing means going back to original goal of investing equal amounts in all stocks and creating a value of 3000

Thus again we need to invest 1000 in each stock and so as we can see from the calculations above, we need to sell A and buy B and C

For A, no of shares should be = 1000/72 = 13.89 and we currently have 14.28 shares thus we will sell 0.39 shares of A to rebalance

For B, no of shares should be = 1000/ 81 = 12.35 and we currently have 11.76 shares, thus we will buy 0.59 shares of B to rebalance

For C, no of shares should be = 1000/98 = 10.2 whereas we have 9.52 shares and thus we will buy 0.72 shares of C to rebalance

Add a comment
Know the answer?
Add Answer to:
Create an equally-weighted index for period 1, w/ initial value of 3000. How many shares of...
Your Answer:

Post as a guest

Your Name:

What's your source?

Earn Coins

Coins can be redeemed for fabulous gifts.

Not the answer you're looking for? Ask your own homework help question. Our experts will answer your question WITHIN MINUTES for Free.
Similar Homework Help Questions
  • Stock P1 Q1 P2 Q2 P3 Q3 A $70 200 $72 200 $37.5 400 B $85...

    Stock P1 Q1 P2 Q2 P3 Q3 A $70 200 $72 200 $37.5 400 B $85 500 $81 500 $83 500 C $105 300 $98 300 $105.5 300 1-What is the value of the price-weighted index for periods 1, 2 and 3? What is the return of the price-weighted index between periods 1 and 2? And between periods 2 and 3?

  • In the following table, Pt represents the price at time t and Qt represents shares outstanding at time t. If the equally...

    In the following table, Pt represents the price at time t and Qt represents shares outstanding at time t. If the equally weighted index of the three stocks at t=0 was 300, what is the index at t=1? Please show calculations. P0 Q0 P1 Q1 A 88 100 93 100 B 45 200 40 200 C 96 200 48 400

  • A value-weighted index consisting of stocks A, B, and C was created yesterday. When the index...

    A value-weighted index consisting of stocks A, B, and C was created yesterday. When the index was created, stocks A,B, and C traded for $80, $45, and $125, respectively. The number of shares outstanding for A,B, and C, was 500, 900, and 600 when the index was formed. Today, stocks A, B, and C trade for $65, $50, and $145, respectively. Find the return on the index from yesterday to today Round intermediate steps and your final answer to four...

  • Consider the three stocks in the following table. Pt represents price at time t, and Qt...

    Consider the three stocks in the following table. Pt represents price at time t, and Qt represents shares outstanding at time t. Stock C splits two-for-one in the last period. P0 Q0 P1 Q1 P2 Q2 A 81 100 86 100 86 100 B 41 200 36 200 36 200 C 82 200 92 200 46 400 Calculate the first-period rates of return on the following indexes of the three stocks: (Do not round intermediate calculations. Round your answers to...

  • 7b. Consider three stocks A, B, and C, with closing price at time t being P, and the number of shares time t as Q S...

    7b. Consider three stocks A, B, and C, with closing price at time t being P, and the number of shares time t as Q Stock C splits three-for-one at the beginning of period -2 (before the market opens). 1000 the market opens of shares outstanding at 43 1000 35 56 102 1000 30 5000 67 5000 5000 B 62 C 98 i. What is the return on a price-weighted index of the three stocks for the period r-0 to...

  • QUESTION 1 A market value weighted index has three stocks in it, call them A, B,...

    QUESTION 1 A market value weighted index has three stocks in it, call them A, B, and C, priced at 32, 58, and 83 per share. Each firm has 455, 143 and 155 thousand shares outstanding, res pective ly. The value of the index at that time is 742. Over the course of the next quarter, the prices of the three stocks change to 40, 82, 55, respectively. What is the new value of the index? Enter answer accurate to...

  • c. Calculate the rate of return for the second period (t 1 to t- 2). Rate...

    c. Calculate the rate of return for the second period (t 1 to t- 2). Rate of return 5 of 12 Next > < Prev Consider the three stocks in the following table. P, represents price at time t, and Q, represents shares outstanding at time t. Stocka C splits two for one in the last period. PO P1 01 P2 02 A 110 500 115 500 115 85 500 B 90 600 85 600 600 1,200 80 600 100...

  • In the following table, P, represents the price at time, and represents shares outstanding at time....

    In the following table, P, represents the price at time, and represents shares outstanding at time. If the equally weighted index of the three stocks att was 300, what is the index at 1? P Q P Q A 88 100 93 100 B 45 200 40 200 C 96 200 48 400 Answer: Show your calculations here. 6. You sell short 250 shares of BNO that are currently selling at $72 per share. You post the 50% margin required...

  • hi..please help..so i took a test and i got all of it wrong and im not...

    hi..please help..so i took a test and i got all of it wrong and im not sure how to do this problems..this is Investment class..please explain every step because im going to study using this material.thank you Fall 2010 4. Compare and contrast open end ve closed-end mutual funds will sell as long you want to buy the share, management comparin thom to you, while closed end mutual funds Preferred stocks on any public traced securities such as cquity, bond...

  • PROBLEM i Create a column of monthly returns for your 2 stocks and the following 3...

    PROBLEM i Create a column of monthly returns for your 2 stocks and the following 3 portfolios. Organize your spreadsheet as follows: a. Date VRSN (#1) Portfolio 1 80% in A 20% in B Portfolio 2 50% in A 50% in B Portfolio 3 20% in A 80% in B MNST (#2) S&P 500 X.x X.x Xx Xx Xx b. Calculate the historical average return and standard deviation for your stocks and the portfolios. Recall you are using historical data,...

ADVERTISEMENT
Free Homework Help App
Download From Google Play
Scan Your Homework
to Get Instant Free Answers
Need Online Homework Help?
Ask a Question
Get Answers For Free
Most questions answered within 3 hours.
ADVERTISEMENT
ADVERTISEMENT
ADVERTISEMENT